2002 AMC 10B Problems/Problem 25

Revision as of 00:55, 17 February 2010 by 506google (talk | contribs) (Created page with '== Problem == When 15 is appended to a list of integers, the mean is increased by 2. When 1 is appended to the enlarged list, the mean of the enlarged list is decreased by 1. How…')
(diff) ← Older revision | Latest revision (diff) | Newer revision → (diff)

Problem

When 15 is appended to a list of integers, the mean is increased by 2. When 1 is appended to the enlarged list, the mean of the enlarged list is decreased by 1. How many integers were in the original list?

$\mathrm{(A) \ } 4\qquad \mathrm{(B) \ } 5\qquad \mathrm{(C) \ } 6\qquad \mathrm{(D) \ } 7\qquad \mathrm{(E) \ } 8$